LSAT and Law School Admissions Forum

Get expert LSAT preparation and law school admissions advice from PowerScore Test Preparation.

 Administrator
PowerScore Staff
  • PowerScore Staff
  • Posts: 8916
  • Joined: Feb 02, 2011
|
#59617
Complete Question Explanation
(The complete setup for this game can be found here: lsat/viewtopic.php?t=26510)

The correct answer choice is (A)

If V marches immediately behind P, then, by adding the first rule we arrive at the following block:

pt43_j04_g1_q3a.png
And, after adding the second rule, we arrive at the following super-block:

pt43_j04_g1_q3b.png
The question asks for which could be the second group, and a quick glance at the templates shows that Template #1 conforms to the block above. However, the group that marches second in Template #1—P—is not one of the answer choices. Hence, we must look at one of the other templates for the answer. Because Templates #3 and #4 cannot conform to the super-block above, the only possible source of the correct answer is Template #2. Because Template #2 features F as the second group, the correct answer to this problem must be F, and answer choice (A) must be correct. However, let’s take a moment to examine this further.

In Template #2, F and V are in groups 2 and 4, respectively. Applying the super-block, we can create the following diagram:

pt43_j04_g1_q3c.png
At this juncture, the only remaining uncertainty is the placement of G and J, which can be placed in group 1 or 5:

pt43_j04_g1_q3d.png
As this hypothetical meets the rules of the game and the condition imposed in question #3, it shows that F can be in the second group.

Answer choice (A): This is the correct answer choice.

Answer choice (B): Under the condition in this question, G must march in group 1 or 5.

Answer choice (C): Under the condition in this question, J must march in group 1 or 5.

Answer choice (D): Under the condition in this question, M must march in group 6.

Answer choice (E): Under the condition in this question, V must march in group 4.
You do not have the required permissions to view the files attached to this post.
 KG!
  • Posts: 69
  • Joined: May 26, 2020
|
#76929
Administrator wrote:Complete Question Explanation
(The complete setup for this game can be found here: https://forum.powerscore.com/lsat/viewtopic.php?t=26510)

The correct answer choice is (A)

If V marches immediately behind P, then, by adding the first rule we arrive at the following block:

pt43_j04_g1_q3a.png
And, after adding the second rule, we arrive at the following super-block:

pt43_j04_g1_q3b.png
The question asks for which could be the second group, and a quick glance at the templates shows that Template #1 conforms to the block above. However, the group that marches second in Template #1—P—is not one of the answer choices. Hence, we must look at one of the other templates for the answer. Because Templates #3 and #4 cannot conform to the super-block above, the only possible source of the correct answer is Template #2. Because Template #2 features F as the second group, the correct answer to this problem must be F, and answer choice (A) must be correct. However, let’s take a moment to examine this further.

In Template #2, F and V are in groups 2 and 4, respectively. Applying the super-block, we can create the following diagram:

pt43_j04_g1_q3c.png
At this juncture, the only remaining uncertainty is the placement of G and J, which can be placed in group 1 or 5:

pt43_j04_g1_q3d.png
As this hypothetical meets the rules of the game and the condition imposed in question #3, it shows that F can be in the second group.

Answer choice (A): This is the correct answer choice.

Answer choice (B): Under the condition in this question, G must march in group 1 or 5.

Answer choice (C): Under the condition in this question, J must march in group 1 or 5.

Answer choice (D): Under the condition in this question, M must march in group 6.

Answer choice (E): Under the condition in this question, V must march in group 4.

HI,
Would it have been acceptable to just look at what answer choices could only fit in slot 2 (FJP) and then quickly eliminate those and then further assess?
Thanks in advance!!
 Adam Tyson
PowerScore Staff
  • PowerScore Staff
  • Posts: 5153
  • Joined: Apr 14, 2011
|
#77029
Yes indeed, KG! Whether you did templates or not, your main diagram should help you quickly eliminate answers B, D, and E, and allow you to just focus on A or C. Testing just one of them will get you the right answer - either the one you test works, and is correct, or it does not work and the other one must be correct. Even without templates, this one should be relatively quick and painless!

Get the most out of your LSAT Prep Plus subscription.

Analyze and track your performance with our Testing and Analytics Package.